Difference between revisions of "1971 AHSME Problems/Problem 5"
(Created page with "Solution 1: Let the measure of P equal (BD-AC)/2. BD = 80. Let the measure of Q equal 1/2 the measure of AC. Add the measures of P and Q to get (80-AC)/2 + AC/2 Therefore,...") |
(added second solution) |
||
(5 intermediate revisions by 2 users not shown) | |||
Line 1: | Line 1: | ||
− | + | == Problem 5 == | |
− | + | Points <math>A,B,Q,D</math>, and <math>C</math> lie on the circle shown and the measures of arcs <math>\widehat{BQ}</math> and <math>\widehat{QD}</math> | |
− | + | are <math>42^\circ</math> and <math>38^\circ</math> respectively. The sum of the measures of angles <math>P</math> and <math>Q</math> is | |
− | + | ||
− | + | <math>\textbf{(A) }80^\circ\qquad | |
+ | \textbf{(B) }62^\circ\qquad | ||
+ | \textbf{(C) }40^\circ\qquad | ||
+ | \textbf{(D) }46^\circ\qquad | ||
+ | \textbf{(E) }\text{None of these} </math> | ||
+ | |||
+ | <asy> | ||
+ | size(3inch); | ||
+ | draw(Circle((1,0),1)); | ||
+ | pair A, B, C, D, P, Q; | ||
+ | P = (-2,0); | ||
+ | B=(sqrt(2)/2+1,sqrt(2)/2); | ||
+ | D=(sqrt(2)/2+1,-sqrt(2)/2); | ||
+ | Q = (2,0); | ||
+ | A = intersectionpoints(Circle((1,0),1),B--P)[1]; | ||
+ | C = intersectionpoints(Circle((1,0),1),D--P)[0]; | ||
+ | draw(B--P--D); | ||
+ | draw(A--Q--C); | ||
+ | label("$A$",A,NW); | ||
+ | label("$B$",B,NE); | ||
+ | label("$C$",C,SW); | ||
+ | label("$D$",D,SE); | ||
+ | label("$P$",P,W); | ||
+ | label("$Q$",Q,E); | ||
+ | //Credit to chezbgone2 for the diagram</asy> | ||
+ | |||
+ | |||
+ | == Solution 1 == | ||
+ | We see that the measure of <math>P</math> equals <math>(\widehat{BD}-\widehat{AC})/2</math>, and that the measure of <math>Q</math> equals <math>\widehat{AC}/2</math>. | ||
+ | Since <math>\widehat{BD} = \widehat{BQ} + \widehat{QD} = 42^{\circ} + 38^{\circ} = 80^{\circ}</math>, the sum of the measures of <math>P</math> and <math>Q</math> is <math>\widehat{BD}/2 = 80^{\circ}/2 = 40^{\circ} \Longrightarrow \boxed{\textbf{(C) }40^{\circ}}</math>. | ||
+ | |||
+ | == Solution 2 == | ||
+ | Arcs are measured by the angle measures of their corresponding [[central angle|central angles]]. Thus, the [[inscribed angle]] <math>\measuredangle BAQ = \tfrac{42^{\circ}}2 = 21^{\circ}</math>, and, likewise, <math>\measuredangle QCD = \tfrac{38^{\circ}}2 = 19^{\circ}</math>. Thus, by [[supplementary]] angles, <math>\measuredangle PAQ = 180^{\circ} - \measuredangle BAQ = 159^{\circ}</math>, and <math>\measuredangle PCQ = 180^{\circ} - \measuredangle QCD = 161^{\circ}</math>. Because the sum of the [[interior angle]] measures of a quadrilateral add to <math>360^{\circ}</math>, we see that <math>\measuredangle P + \measuredangle Q = 360^{\circ} - \measuredangle PAQ - \measuredangle PCQ = 360^{\circ} - 159^{\circ} - 161^{\circ} = 40^{\circ}</math>. Thus, our answer is <math>\boxed{\textbf{(C) }40^{\circ}}</math>. | ||
+ | |||
+ | == See Also == | ||
+ | {{AHSME 35p box|year=1971|num-b=4|num-a=6}} | ||
+ | {{MAA Notice}} | ||
+ | [[Category:Introductory Geometry Problems]] |
Latest revision as of 09:03, 1 August 2024
Contents
Problem 5
Points , and lie on the circle shown and the measures of arcs and are and respectively. The sum of the measures of angles and is
Solution 1
We see that the measure of equals , and that the measure of equals . Since , the sum of the measures of and is .
Solution 2
Arcs are measured by the angle measures of their corresponding central angles. Thus, the inscribed angle , and, likewise, . Thus, by supplementary angles, , and . Because the sum of the interior angle measures of a quadrilateral add to , we see that . Thus, our answer is .
See Also
1971 AHSC (Problems • Answer Key • Resources) | ||
Preceded by Problem 4 |
Followed by Problem 6 | |
1 • 2 • 3 • 4 • 5 • 6 • 7 • 8 • 9 • 10 • 11 • 12 • 13 • 14 • 15 • 16 • 17 • 18 • 19 • 20 • 21 • 22 • 23 • 24 • 25 • 26 • 27 • 28 • 29 • 30 • 31 • 32 • 33 • 34 • 35 | ||
All AHSME Problems and Solutions |
The problems on this page are copyrighted by the Mathematical Association of America's American Mathematics Competitions.